blob: 27551d48d4db38f08658d226e143a6a51ac43e6d (
plain)
1
2
3
4
5
6
7
8
9
10
11
12
13
14
15
16
17
18
19
20
21
22
23
24
25
26
27
28
29
30
31
32
33
34
35
36
37
38
39
40
41
42
43
44
45
46
47
48
49
50
51
52
53
54
55
56
57
58
59
60
61
62
63
64
65
66
67
68
69
70
71
72
73
74
75
76
77
78
79
80
81
82
83
84
85
86
87
88
89
90
91
92
93
94
95
96
97
98
99
100
|
% SPDX-License-Identifier: CC-BY-SA-4.0
%
% Copyright (c) 2020 Philipp Le
%
% Except where otherwise noted, this work is licensed under a
% Creative Commons Attribution-ShareAlike 4.0 License.
%
% Please find the full copy of the licence at:
% https://creativecommons.org/licenses/by-sa/4.0/legalcode
\phantomsection
\addcontentsline{toc}{section}{Exercise 4}
\section*{Exercise 4}
%%%%%%%%%%%%%%%%%%%%%%%%%%%%%%%%%%%%%%%%%%%%%%%%%%%%%%%%%%%%%%%%%%%%%%%%%%%%%%%
\begin{question}[subtitle={Sampling Periodic Signals}]
\begin{equation*}
u(t) = \SI{2}{V} \cos\left(2\pi \SI{2}{MHz} t + \SI{60}{\degree}\right)
\end{equation*}
The signal is sampled with a sampling period of $T_S = \SI{125}{\nano\second}$. The first sample taken is $u(t = 0)$.
\begin{tasks}
\task
Plot the function from $t = 0$ to $t = \SI{1}{\micro\second}$!
\task
Calculate the samples $n = 0 \dots 8$!
\task
What is the DTFT of the signal?
Hints:
\begin{equation*}
\begin{split}
x[n] = e^{-j a n} &= \underline{X}_{\frac{2\pi}{T_S}}\left(e^{-j T_S \omega}\right) = 2 \pi \cdot \delta \left(\omega + a\right) \\
\cos\left(b\right) &= \frac{1}{2} \left(e^{j b} + e^{-j b}\right)
\end{split}
\end{equation*}
\task
Can the DFT directly applied to the signal? If yes, determine the smallest $N$ and give the values of all $\underline{U}[k]$!
\task
What is the longest possible sampling period? What must be considered at this sampling period?
\task
Now, the sampling period is changed to $T_S = \SI{0.5}{\micro\second}$. There is no anti-aliasing filter. The reconstruction filter is an ideal low-pass filter with a cut-off frequency of \SI{50}{kHz}. Give the reconstructed output function in the time domain! Give an explanation in the frequency domain!
\end{tasks}
\end{question}
\begin{solution}
\begin{tasks}
\end{tasks}
\end{solution}
%%%%%%%%%%%%%%%%%%%%%%%%%%%%%%%%%%%%%%%%%%%%%%%%%%%%%%%%%%%%%%%%%%%%%%%%%%%%%%%
%\begin{question}[subtitle={Sampling Non-Periodic Signals}]
% \begin{tasks}
% \end{tasks}
%\end{question}
%
%\begin{solution}
% \begin{tasks}
% \end{tasks}
%\end{solution}
%%%%%%%%%%%%%%%%%%%%%%%%%%%%%%%%%%%%%%%%%%%%%%%%%%%%%%%%%%%%%%%%%%%%%%%%%%%%%%%
\begin{question}[subtitle={Quantization}]
The signal of task 1b) is now quantized. The quantizer has $8$ discrete values. These values are equally distributed between \SI{-2}{V} and \SI{2}{V}. Prior to sampling, the original time-continuous signal passed through an ideal low-pass filter with a cut-off frequency of \SI{4}{MHz}.
\begin{tasks}
\task
Define a mapping from the value-continuous samples to the value-discrete samples!
\task
The value-discrete samples are now pulse-code modulated. How many bits are required?
\task
Determine the quantization error for each value-discrete sample! How much is the signal-to-noise ratio?
\task
3 bits are a very poor resolution. How many bits are appropriate for the quantizer to obtain the best signal-to-noise ratio? Effects of the window filter are neglected. Assume that the signal has passed through a processing chain with a total gain of \SI{25}{dB} and noise figure of \SI{12}{dB} prior to quantization. The input of the quantizer has an impedance of \SI{50}{\ohm}. % 14 bits
\end{tasks}
\end{question}
\begin{solution}
\begin{tasks}
\end{tasks}
\end{solution}
%%%%%%%%%%%%%%%%%%%%%%%%%%%%%%%%%%%%%%%%%%%%%%%%%%%%%%%%%%%%%%%%%%%%%%%%%%%%%%%
%\begin{question}[subtitle={Decibel}]
% \begin{tasks}
% \end{tasks}
%\end{question}
%
%\begin{solution}
% \begin{tasks}
% \end{tasks}
%\end{solution}
|